Comprehensive coverage

A quantum threat to special relativity / David Z. Albert and Rebecca Gal Chen

Quantum entanglement, like many other quantum effects, goes against some of the deepest intuitions we have about the world. It may also undermine Einstein's special theory of relativity

By David Z. Albert and Rebecca Gal Chen

Intuition has always told us that if we want to move a rock, for example, we must touch the rock, or touch a stick that touches the rock, or obey a command that passes through vibrations in the air to the ear of a person with a stick who can push the rock, or any other series like this. More generally, this intuition says that things can only directly affect things that are actually by them. If A affects B without actually being by him, then the effect in question must exist indirect - There must be something that is transmitted through a chain of events in which each event directly creates the event that follows it, a chain that covers without skipping the entire distance between A and B. Every time we think we find an exception to this intuition - for example, lifting a switch that turns on the street lights in the city (but then we realize that the action takes place through cables) or listening to a radio transmission (but then we realize that the radio waves move through space) - it turns out that we actually don't We found an exception. That is, at least not in our day-to-day experience in the world.

We call this intuition the term "locality".

Quantum mechanics has shattered many intuitions, but this is the most profound of them all. And this particular shattering carries in its wings a threat whose shadow has not yet been removed, a threat to special relativity, one of the cornerstones of our 21st century physics.

The thing from outer space

The curvature of space-time according to Einstein's equations. Image: NASA
The curvature of space-time according to Einstein's equations. Illustration: NASA

Let's go back a bit. Before quantum mechanics came into the world, and in fact at the very beginning of the scientific study of nature, at that time in the distant past, scholars believed that it was basically possible to get a complete description of the physical world if we described each and every one of the smallest and most fundamental physical components of the world. The complete story of the world can be expressed as the sum of the component stories.

Quantum mechanics put an end to this belief.

Measurable and actual physical properties of collections of particles may, unequivocally and clearly, exceed, miss, or be completely different from the sum of the properties of the individual particles. For example, according to quantum mechanics, it is possible to arrange a pair of particles so that the distance between them is exactly two meters, and yet, none of the particles will have a definite position. Furthermore, the accepted approach to understanding quantum physics, known as the Copenhagen interpretation - which was formulated by the great Danish physicist Niels Bohr at the beginning of the last century and has since been handed down from professor to student over the course of several generations - stubbornly claims that it is wrong to say that we do not know the facts about the exact location of individual particles, but simply Not at all Such facts. The question of what is the position of a single particle will be as meaningless as, for example, the question of whether the number five is married. The problem is not epistemological (that is, dealing with what we know) but ontological (dealing with what exists).

The professional term physicists use to describe particles that are bound together in the way described here is quantum entanglement. The entangled property need not be position: two particles can spin in opposite directions, yet neither of them is the one spinning clockwise. Or exactly one of the two particles will be in an excited state, but neither of them is clearly the excited particle. The entanglement can link particles regardless of where they are, what they are, and what forces they may exert on each other. Basically, they could well be an electron and a neutron standing at two opposite ends of the galaxy. The interweaving therefore brings to the world a kind of intimacy in the material, which they did not dare to dream of before.

The interweaving stands in the infrastructure of new fields of research that hold growing promise: quantum computing and quantum encryption, which can provide us with the ability to solve certain problems that are outside the practical range of a normal computer and the ability to communicate with complete security against eavesdropping.

But it seems that the interweaving also entails another phenomenon, shrouded in deep mystery and sharply counterintuitive: a phenomenon called "nonlocality", the possibility of physically influencing something without touching it and without touching any series of entities that extends from here to there. According to non-locality, a punch in Tel Aviv can crush a face in Jerusalem without affecting any other physical entity (no air molecule, no electron in an electric wire, no glimmer of light) in the space between them.

The most alarming thing about non-locality, apart from the jarring strangeness inherent in it, is that this feature carries with it a threat to the special theory of relativity as we know it today. In recent years, this concern, which has finally gained entry into the hall of serious thoughts in the field of physics, has become the focus of discussions that may, in the end, tattoo, distort, reimagine, consolidate or disintegrate the very foundations of physics.

Extreme rewritings of reality

Quantum mechanics disturbed Einstein's rest in countless ways. The over-quoted concern about its randomness ("God doesn't play dice") is just one of them. However, the only objection he formally expressed and bothered to write a paper about, concerned the strangeness of quantum entanglement. This objection lies at the heart of the argument known as EPR, named after its three authors, Einstein and his colleagues Boris Podolsky and Nathan Rosen. In their 1935 paper, "Can the description of physical reality provided by quantum mechanics be considered complete?" They answer the question they raised with a firm and well-founded "no".

Their argument mainly used one particular instruction from the recipe, or mathematical algorithm, of quantum mechanics used to predict the outcomes of experiments. Suppose we measure the position of a particle that is quantum entangled with another particle, so that neither has an exact position, as we mentioned here. Obviously, when we know what the result of the measurement is, the description of the first particle will change, because then we will know where it was at that moment. But the algorithm also tells us to change the description we gave to the second particle, and change it immediately, and it doesn't matter how far it is from the first particle, or what is in between.

The entanglement was an indisputable fact in the picture of the world that quantum mechanics gave physicists, but no one before Einstein thought about the consequences of this fact. To him, the interweaving was not just something strange, but something questionable. He felt that she was something out of this world. She looked, mostly, non-local.

In those days no one was willing to take into account the possibility that there is real physical non-locality in the world, not Einstein, not Bohr, not a single one. Einstein, Podolsky and Rosen assumed as a matter of course in their article that the apparent non-locality of quantum mechanics is only apparent, that it must be some kind of mathematical anomaly or a faulty notation method, or at least some product of the algorithm that can be gotten rid of. There was no doubt in their minds that it was possible to concoct predictions for quantum mechanics experiments without resorting to non-local steps.

In their paper they presented an argument that said, more or less, that if there is no actual non-locality in the world (as everyone assumed), and if the experimental predictions of quantum mechanics are correct, then quantum mechanics does not include certain aspects of the world. There must be parts of the world story that she leaves out.

Bohr responded to the EPR article overnight. He feverishly composed a rebuttal letter that did not deal with any of the solid scientific arguments in the article, but rather addressed, in a vague and sometimes enigmatic style, the way in which the authors used the word "reality" and their definition of "components of physical reality". He spoke at length about the distinction between object and subject, the conditions that must exist for the questions asked to be meaningful, and the nature of human language. Science needs, according to Bohr, a "radical rewriting of our approach to physical reality."

Even so, Bohr went out of his way to agree with EPR's paper on one point: it is clear as day that actual physical non-locality cannot exist. The apparent non-locality, according to him, is simply another reason why we must abandon the old-fashioned aspiration, which has lost steam but was evident from every letter in the EPR article, to be able to read from the equations of quantum mechanics a realistic picture of the world, a picture of what really exists in front of us in every A moment and a moment. Bohr insisted, in fact, that not only are we looking at the world through a cloudy glass, but also that this unclear and dim picture is the most real thing there is.

Bohr's response was an intriguing philosophical response to a distinctly scientific concern. But what was more intriguing than that was that Bohr's reaction was sanctified and became the official creed of theoretical physics. From then on, spending time thinking about these issues was considered a form of heresy. The physicist community thus turned its back on its old ambitions, to remove the lot from the real face of the world, and for a long time thereafter turned metaphysical questions in search of respect for the fiction of fantasy.

Even today this crucial part of Einstein's legacy remains in the shadows. Walter Isaacson's best-selling 2007 biography of Einstein reassures the reader that Einstein's critique of quantum mechanics has since been shelved. And that is not true.

The return of the repressed

The first serious scientific engagement with the EPR argument appeared (after 30 years of more or less complete neglect) in a famous 1964 paper written by the extraordinary Irish physicist John S. Bell. Bell's work shows that Bohr was wrong in assuming that he was not wrong in his understanding of quantum mechanics and that Einstein was wrong in the question what There was the mistake in Bohr's understanding. In order to internalize what the mistake actually was, it is necessary to abandon the idea of ​​locality.

The crucial question is whether the non-locality, which at least seems to appear in the algorithm of quantum mechanics, is only apparent or something beyond that. Bell seems to be the first person to ask himself what exactly this question means. What can distinguish real physical non-locality from apparent non-locality? He concluded that if there exists an algorithm whose locality is clear and unambiguous, and whose predictions for the results of experiments are the same as the predictions of the quantum mechanics algorithm, then Einstein and Bohr were right when they dismissed the non-locality of quantum mechanics as a product of that particular formalism and nothing more. Conversely, if no algorithm can avoid non-locality, then non-locality must be an actual physical phenomenon. Bell analyzed a particular weaving scenario and concluded that there is no mathematical possibility to build such a local algorithm.

And therefore, the actual physical world is non-local. point.

This conclusion turns everything upside down. Einstein, Bohr and all the others have always taken for granted that any real conflict between quantum mechanics and the principle of locality is simply bad news for quantum mechanics. But Bell has now shown that nonlocality conflicts not only with the abstract theoretical structure of quantum mechanics, but also with certain experimental predictions of it. Experiments, especially work done by Alain Aspe and his colleagues at the Institute of Optics in Plessaud, France, in 1981 and later, have shown beyond any doubt that these predictions are indeed correct. The bad news was therefore not the lot of quantum mechanics but of the principle of locality, and anyway, it seems, of special relativity, since, at least apparently, it is built on the assumption of locality.

A metaphysical mystery tour

The main reaction to Bell's work, a reaction still prevalent in many quarters even today, was even more obscurity. Bell showed that any theory capable of reproducing the experimental predictions of quantum mechanics for entangled pairs of particles, including quantum mechanics itself, must include actual physical nonlocality.

Since then, this message has been practically ignored. Instead, almost everyone says, Bell has shown that any theory that attempts to replace the world picture of orthodox quantum mechanics with something more in line with our classical metaphysical expectations—any theory that includes determinism, philosophical realism, or hidden variables—must be nonlocal, if it is able to reproduce the predictions of quantum mechanics for the EPR system [in the box on the right you can see two ways to escape Bell's conclusion]. The people did read Bell's work, but as if a crooked mirror road.

Only a small minority of physicists managed to avoid this particular misunderstanding and to perceive that the meaning of Bell's proof and Aspa's experiments was that the world itself was found to be non-local, but even they, almost without exception, believed that this non-locality posed no threat Special on special relativity.

This explanation stems from the concept that special relativity involves an inseparable connection with the impossibility of transmitting messages at a speed higher than the speed of light. After all, if the special theory of relativity is correct, it can be argued that no material message carrier can accelerate from rest to speeds higher than the speed of light. And it is possible to argue that a message that is transmitted faster than light will, according to certain clocks, be a message that arrives before it is sent, a claim that has the power to arouse all the demons of the paradoxes of time travel.

Already in 1932, the brilliant Hungarian mathematician John von Neumann proved that it is impossible to manipulate the non-locality of quantum mechanics and use it to build a mechanism for transmitting messages in an instant. For decades, the entire physics community saw von Neumann's proof as a kind of insurance certificate that the non-locality of quantum mechanics and special relativity could coexist peacefully.

The non-local experience for the issue

It took another 30 years after Bell's paper was published before physicists dared to look directly at these issues. The first discussion of quantum non-locality and relativity blessed with honesty, uncompromising, logical completeness, uniform level and clarity, appeared in 1994 in a book written by Tim Modlin of Rutgers University, called "Quantum non-locality and relativity". Modlin emphasized in his work that the question of settlement between non-locality and special relativity is much more elusive and subtle than the impression created by worn-out passwords based on instant messages.

Modlin's work was published against the background of a new and profound change in the intellectual environment, which began in the early 80s and continues to this day. The validity of the conviction in Bohr's position, that it is impossible for the subatomic world to have a realistic and philosophically conservative description, began to be tangibly undermined everywhere. During this period it was already possible to find several promising and clear scientific proposals that provide a description of this kind, at least in an approximation that neglects the effects of special relativity. Proposals included the mechanics developed by England's David Boehm in the early 50s (which inspired Bell's work, but was otherwise generally ignored) and the GRW model by Giancarlo Girardi, Abelardo Rimini and Tullio Weber of Italy. The long-standing ambitions of physics, to lead the way to metaphysics, to tell us in an explicit and direct way how the world actually looks, ambitions that had been abandoned and rested, dormant, for more than 50 years, began, slowly, to reawaken.

Modlin's book focused on three important matters. First, special relativity is an assertion about the geometric nature of space and time. It claims that it is impossible to transfer mass or energy or information or causal effects faster than light. But none of these requirements, on its own, even comes close to guaranteeing that the theory's claim about the geometry of space and time is a true claim. Thus, von Neumann's proof of message transmission gives us no guarantee that quantum non-locality and special relativity can coexist peacefully.

Second, the truth of special relativity can, in fact, be reconciled without any problem with a vast array of hypothetical mechanisms for the light-fast transfer of mass, energy, information, and causal effects. In the 60s, for example, Gerald Feinberg of Columbia published a relativistic theory completely without internal contradictions of a hypothetical class of particles, which he called tachyons, for which it is never possible to travel slower from the light Modlin came up with more examples.

Therefore, the mere existence of non-locality in quantum mechanics does not necessarily mean that quantum mechanics cannot exist alongside special relativity. So maybe there is hope.

However, as Modlin emphasized in the third point he raised, the particular type of action at a distance that we encounter in quantum mechanics is a completely different animal from the type that appeared in Feinberg's tachyons or in Modlin's other examples. The non-local effect of quantum particles on each other seems supernatural because it does not depend on the spatial arrangement of the particles or the physical properties inherent in them - in contrast to all the relativistic effects mentioned in the previous paragraphs - but only on the question of whether or not the particles in question are quantum entwined with each other.

It seems that the type of non-locality encountered in quantum mechanics requires absolute simultaneity, which poses a real and deadly threat to special relativity.

And that's the trouble.

Hope for special relativity?

Two new results, which curiously pull in different directions, have emerged from this debate over the past few years. The first suggests a way in which quantum non-locality will come into line with special relativity; while the second reveals a new blow that our deepest intuitions about the world absorb from the combination of quantum mechanics and special relativity.

The first result appeared in 2006 in a remarkable paper by Roderich Tomolka, a young German mathematician now researching at Rutgers. Tomolka showed how all the experimental predictions of quantum mechanics could be reproduced for pairs of entangled particles by a sophisticated rewriting of GRW theory (remember, this theory offers a philosophically realistic way to obtain the predictions of quantum mechanics under many conditions). The rewrite is non-local, but fully consistent with the space-time geometry of special relativity.

This work is still very much in its infancy. No one has yet been able to write a satisfactory version of Tumolka's theory that can be applied to particles that attract or repel each other. More than that, his theory introduces a new type of non-locality into the laws of nature - non-locality not only in space but also in time! To use this theory to determine the probabilities of what should happen, it is not enough to enter the current physical state in full (as is customary to do in physical theories), but certain facts about the past must also be entered. This characteristic and others raise concerns, but there is no doubt that Tumolka removed part of the basis for Modlin's fear that the non-locality of quantum mechanics could not coexist peacefully with special relativity.

The second result recently obtained, and discovered by one of us (Albert), shows that combining quantum mechanics and special relativity requires us to give up another conviction that has been ingrained in us since time immemorial. We believe that everything that needs to be said about the world can be cast in the form of a narrative, or a story. Or, in more technical and precise terms: everything that needs to be said can be packaged in a final set of verses of the form "in time t1This The exact physical state of the world" and "in time t2 זה the exact physical state of the world,” and so on. But the phenomenon of quantum entanglement and the space-time geometry of relativity, taken together, show that the physical history of the world is far too rich for us to do so.

The trouble is that special relativity tends to mix space and time in a way that converts the quantum entanglement between distinct physical systems and turns it into something similar to the entanglement between physical systems at different times - something that unequivocally and clearly exceeds, eludes, or is completely different from any possible sum of states at distinct moments of time.

The result, like most theoretical results in quantum mechanics, involves dealing with a mathematical entity called a wave function, a concept formulated by Erwin Schrödinger 80 years ago to define quantum states. A wave function is the thing from which physicists deduce the possibility (in fact, the necessity) of entanglement, of particles with infinite positions and so on. And it is the wave function that lies at the heart of the puzzles about the non-local effects of quantum mechanics.

but what is she Exactly, this wave function? A fierce debate is now raging on this issue among researchers of the fundamentals of physics. Is a wave function an actual physical object, or is it something similar to a law of motion or an internal property of particles or a relationship between points in space? Or maybe it's just the current information we have about the particles? or what?

It is impossible to represent the wave functions of quantum mechanics mathematically using less than a space of an unimaginable number of dimensions, called configuration space. If, as some people claim, wave functions are to be seen as actual physical objects, then we must take seriously the idea that the history of the world unfolds not in the three-dimensional space of our everyday experience or in the four-dimensional space of special relativity, but in the vast and infinite configurational space This familiar, from which the illusion of XNUMXD arises in some way. Our concept of three-dimensional locality will also need to be understood as an emergent concept. The non-locality of quantum mechanics may be our window to that deeper level of reality.

The state of special relativity, only a little more than a century after it appeared on the world stage, suddenly became a wide open and rapidly developing question. This situation has arisen because physicists and philosophers have finally followed the rough edges of that forgotten debate of Einstein's with quantum mechanics - further ironic proof of Einstein's genius. It is quite possible that the late Guru was wrong exactly where he thought he was right and right exactly where he thought he was wrong. In fact, we may be seeing the world through a glass that is not so dim, contrary to the stubborn opinion that has been ruling the dome for a long, long time.

100 תגובות

  1. I don't understand the meaning of "at the same time" what experiment can confirm that the collapse in the two particles far from each other happened at the same time?
    Is simultaneity for every theoretical observer?

  2. Alon,
    Regarding the response "a believing Jew,
    The question of whether intelligence can be attributed to a particle is a very interesting question and very related to how you define things."
    Regarding the intelligence of humans, animals or inanimate objects you can learn from the definition in computer science
    Artificial intelligence has several definitions and one of the main ones is the ability of an agent to make rational decisions to achieve its goal.

    When we produce a product, a car for example.
    There is a lot of intelligence in the way it works to achieve its goals (burning the fuel in the engine, the steering mechanism, etc.) This does not mean that the vehicle is intelligent!
    Only where the intelligence of the one who created it is expressed.
    True intelligence exists only in those who are able to make decisions. There is law in nature, in this law there is a lot of wisdom, but this does not mean that the objects subject to this law are wise (for example the particle).
    Even a person who is an intelligent creature and is able to make intelligent decisions has wisdom beyond his own wisdom and intelligence.
    And this is the wisdom of the way it was created (the structure of the mind, the eye, the digestive system, the production of proteins, etc...) that we are researching and discovering more and more about, but they are still not in our full reach and probably never will be.

    This is why it is said in the Psalms about the Creator, "How many are your works, O Lord, all of which you have done wisely."
    There is wonderful wisdom in every field of nature ("nature" = God in gematria) it testifies to the wisdom of the one who created it.
    Just as the car or the plane testifies to the wisdom of the person who created it.

    There is the complete discussion on the subject of evolution, but this is not the place because wisdom is not limited only to the field of biology... and in this field, the possibility that so many particles will coincidentally line up and allow life in the given conditions on our planet is absolutely nil, mathematics requires a deliberate hand.

    We need to keep an open and objective mind, understand that there are things beyond our understanding.
    Science and logic require the reality of the Creator

  3. First, there is no limit to the hidden variables - there is a limit to the quantum states that can be described with them.

    And the idea described in the quote is a computational limit that is inferred. As with the finite amount of images an LCD can produce.
    You can always represent any number in one way or another. What is important is not the representation, but the ability to show what the error (or accuracy) is.
    It's just like saying that every particle in the universe represents a bit on a normal computer. If there is a finite number of particles - then I can predict in advance the accuracy I will have in representing a number with a given representation method.
    That is, the way in which accuracy is usually considered is by a priori calculation, not by experiment.

    If we still take your statement:
    "This means that if we manage to use a quantum computer to perform this calculation, it contradicts the theory of hidden variables."
    So a practical description of your words is to build a quantum computer from the entire universe, then try to do calculations on it, and see that its computational sensitivity is greater than 10,000 digits. And if so, there is a contradiction.

    If that is your intention - then yes, it will be possible to contradict that way. This is simply an impractical direction, and is a thought experiment only.

    This is of course only true based on the statement you quoted.
    By the way, the claim that with a normal quantum machine it is possible to produce higher accuracy seems suspicious to me. It seems to me that if you take into account the uncertainty of knowing quantum states - there will be a similar limitation, only that it is more difficult to show. - like the difficulty of setting a screen resolution for CRT monitors.

    In any case, if you are more interested in the topic of arguments against "Bohem" you should look for things in the style of the link I put in the previous message.

    But also remember that in terms of predicting results most of the variants are identical to Copenhagen.

  4. oak,
    I may have misunderstood the limitation of the hidden variables
    When they write:
    A nonlocal theory of this sort predicts that a quantum computer encounters fundamental obstacles when it tries to factor numbers of approximately 10,000 digits or more; an achievable task in quantum mechanics

    So this means that if the theory of hidden variables is correct, we should not be able to factor it into a number with 10000 digits, again because there is no number of memory cells in the visible universe. This means that if we manage to use a quantum computer to perform this calculation, then it contradicts the theory of hidden variables.

    The problem is that some memory cell has to contain the hidden variable, so I tried to think of an experiment of enormous dimensions in which we would at once change several properties for a huge and crazy number of conjugated electrons (spin, momentum and what not), so that the amount of hidden variables that would be required to change the conjugated electrons would increase beyond what that the visible universe enables and creates the contradiction against quantum mechanics.

    By the way, I really hope that I don't make physicists vomit because of the examples I gave, so I am careful and only ask if what I described here, derived from my understanding of the quote from Wikipedia, does indeed challenge the hidden variables?

  5. I learned my lesson - from now on I will avoid government links...

    Reporter:
    "Given a sufficiently large quantum computer, which can be built (or it will automatically be possible to build it), the visible universe itself cannot provide enough "memory cells" for the existence of hidden variables."

    No. The universe does not contain enough particles for calculations to be accurate to 10,000 digits.

    Think of it this way: every pixel on your screen is a particle in the universe. The screen is the whole universe (a bit poetic...). Every color that a pixel can produce is a possible state that can be measured. Now - the number of possible combinations of different screen images is the number of pixels, multiplied by the number of possible colors per pixel.
    Similarly there is a finite measurable amount of quantum states for the universe.

    The statement in the quote you gave does not strengthen or weaken any aspect of the theory, but only concludes this finality.

    For the sake of illustration, let's take the analogy of a screen a step further (any analogy is problematic, but let's try):
    You look at a computer screen, and the image is blurry.
    One theory says you are looking at a CRT (Copenhagen analog) screen.
    A second theory says it's an LCD screen, but you just need glasses or there's baking paper on the screen. (analogous to "Bohem").

    If we use the second theory, it is easy to see why there is a finite number of possibilities.
    In the first theory you have a free "phase", from which theoretically more information can be extracted - but it is said with caution that it is very dependent on the uncertainty of the measurement.

    In short, the statement is only analogous to there being a finite number of possible images, given an LCD screen. There is no support for LCD or CRT…

    I did not understand your statement:
    "According to the theory of hidden variables, it will be possible to make a change in one particle without a reaction in the particle conjugated to it."
    I would appreciate it if you could explain to me why this is due to the theory of hidden variables.

  6. Alon, my father had to work hard to approve your posts. Not pretty like that, before the holiday.

    I think the limitation is not "just" practical but fundamental because it says that, given a large enough quantum computer that can be built (or it will automatically be possible to build it), the visible universe itself cannot provide enough "memory cells" for the existence of hidden variables. It seems to me that this is a fundamental problem because then, according to the theory of hidden variables, it will be possible to make a change in one particle without a reaction in the particle conjugated to it, and this will be a contradiction to the predicted result of quantum mechanics. Beyond that, it is difficult to accept a theory that is practically blocked and the way to break through the block is another assumption of an infinite universe and if I am not mistaken, this is not the assumption nowadays

    By the way, if we build such a computer, maybe then we can finally break the universe and get a real blue screen, like that in the whole sky and get out of the matrix

  7. Sorry that it turns out that I am posting a seventh comment in a row, but now I have finally freed up the topic of intelligence.

    The point of view I have brought about intelligence is not based on the principle of uncertainty, or related in any way to free will. In fact, those who take this approach are usually people who train in absolute determinism,
    (when the meaning of free will already depends on the definitions, so we will ignore this concept if possible).

    Usually these are people who believe in: "God of nature" or "Tao" or Einstein's God - etc.
    That is, it is a point of view that believes that there is "intelligence" or "consciousness" that can be attributed to the universe, and in some way everything that is made up of it - has a part of this "consciousness".
    The more formalists in this point of view, explain it in a similar way to what I did.
    That is:
    Where will you draw the line? mouse? A fish? Cockroach? A bacterium?
    -> The tendency of those with this point of view is to say that it is decreasing. And that there is no yes/no, these are degrees of gray, because there is no limit that can be set.

    Part of their formalism/excuse is to associate intelligence with existence itself, and more specifically with the properties that enable existence.

    It's an interesting point of view that I'm trying to represent, but I don't have a firm opinion one way or the other. It's just interesting sometimes to look at things from other perspectives.

  8. *(problematic link)

    In short, the "Bohm" theory did not show an advantage in prediction over "Copenhagen", but it cannot be said that it was contradicted either.

    Many of those who prefer it, do so for reasons similar to mine - a more coherent point of view - which of course is already something individual.
    And I think that because the theory is "out of order", there is a tendency to see it as more delusional.

  9. But it is no coincidence that this is not officially accepted as a contradiction to the theory... firstly there are variants on the theory, secondly - as we said - it is always better to take such things in a careful context.

    In the same way, the so-called "Bohemian" has the advantage of being able to adapt to relationships, link:

  10. But it is no coincidence that this is not officially accepted as a contradiction to the theory... firstly there are variants on the theory, secondly - as we said - it is always better to take such things in a careful context.

    In the same way, the so-called "Bohemian" has the advantage of being able to adapt to relationships, link:
    http://xxx.lanl.gov/abs/0811.1905

  11. —————————————————————————————————————————-

    The predictions of Copenhagen and "Bohm" intersect in such a strong way that as long as there are no differences, they can be viewed as different descriptions of the same phenomenon.

    However, there is a claim that they managed to find an effect that would give an indication of the existence of the hidden variables (as hinted in the second part that may exist).
    The effect is related to the experiment of photons in two slits, and its results should contradict "Bohm's" prediction. Link:
    http://arxiv.org/abs/quant-ph/0206196

  12. The predictions of Copenhagen and "Bohm" intersect in such a strong way that as long as there are no differences, they can be viewed as different descriptions of the same phenomenon.

    However, there is a claim that they managed to find an effect that would give an indication of the existence of the hidden variables (as hinted in the second part that may exist).
    The effect is related to the experiment of photons in two slits, and its results should contradict "Bohm's" prediction. Link:
    http://arxiv.org/abs/quant-ph/0206196

    But it is no coincidence that this is not officially accepted as a contradiction to the theory... firstly there are variants on the theory, secondly - as we said - it is always better to take such things in a careful context.

    In the same way, the so-called "Bohemian" has the advantage of being able to adapt to relationships, link:
    http://xxx.lanl.gov/abs/0811.1905

  13. The third part talks about computational practicality of the theory. In general, there is a calculation that requires assumptions such as a finite size for the universe.

    The fourth part talks about meanings that arise for quantum computers. I assume that you use the assumption that the universe is finite, and when you take into account the formulas described in the third part, you get that there is a theoretical limit to the accuracy that can be obtained (note - the third part assumed a finite size in order for the calculations to be finite; this part concludes that if the size of the universe is finite, then a computational solution can accurately solve , and therefore there will be a limit to the theoretical accuracy).

    That is, to your question: the universe sets a limit - but not to the correctness of the theory of hidden variables. Given that the description of hidden variables is correct, the universe sets a limit to the precision that can be calculated with a theoretical quantum computer.
    To illustrate - even classical theoretical computers are limited in all kinds of parameters given a finite size to the universe.

    That is, there is no argument for or against here, there are inferences about the computational limits.
    If someone really wants a computer the size of the universe to be able to do calculations with an accuracy of over 10,000 digits, then they have to hope that there are no hidden variables, but that there really is inherent randomness.

  14. interesting. It turns out that I used a forbidden word - which I really don't know what it is. More from yesterday at 14:30...
    I have to post in parts, so that I can understand what...
    ————————————————————————————————-
    Shmulik,
    There are several parts to the paragraph you quoted.

    The first part says that for many scientists the theory is insufficient.
    This is true, and there are several reasons for this, but they are not presented below. At the end of the response I will add what does cause skepticism.

    The second part is from a sentence that, paraphrased, says - if there is no direct indication of the existence of hidden variables - their existence can remain philosophical - but it is possible that a possibility will be found to get an indication of their existence.

  15. Shmulik,
    I have a problem sending more from yesterday - at 14:30 pm.
    I guess we had a "silence" - courtesy of Ben David...
    First - copy-paste the question to what I wrote:
    ————————————————————————————————————
    Shmulik,
    There are several parts to the paragraph you quoted.

    The first part says that for many scientists the theory is insufficient.
    This is true, and there are several reasons for this, but they are not presented below. At the end of the response I will add what does cause skepticism.

    The second part is from a sentence that, paraphrased, says - if there is no direct indication of the existence of hidden variables - their existence can remain philosophical - but it is possible that a possibility will be found to get an indication of their existence.

    The third part talks about computational practicality of the theory. In general, there is a calculation that requires assumptions such as a finite size for the universe.

    The fourth part talks about meanings that arise for quantum computers. I assume that you use the assumption that the universe is finite, and when you take into account the formulas described in the third part, you get that there is a theoretical limit to the accuracy that can be obtained (note - the third part assumed a finite size in order for the calculations to be finite; this part concludes that if the size of the universe is finite, then a computational solution can accurately solve , and therefore there will be a limit to the theoretical accuracy).

    That is, to your question: the universe sets a limit - but not to the correctness of the theory of hidden variables. Given that the description of hidden variables is correct, the universe sets a limit to the precision that can be calculated with a theoretical quantum computer.
    To illustrate - even classical theoretical computers are limited in all kinds of parameters given a finite size to the universe.

    That is, there is no argument for or against here, there are inferences about the computational limits.
    If someone really wants a computer the size of the universe to be able to do calculations with an accuracy of over 10,000 digits, then they have to hope that there are no hidden variables, but that there really is inherent randomness.

    ———————————————————————————————————————————————–

    The predictions of Copenhagen and "Bohm" intersect in such a strong way that as long as there are no differences, they can be viewed as different descriptions of the same phenomenon.

    However, there is a claim that they managed to find an effect that would give an indication of the existence of the hidden variables (as hinted in the second part that may exist).
    The effect is related to the experiment of photons in two slits, and its results should contradict "Bohm's" prediction. Link:
    http://arxiv.org/abs/quant-ph/0206196

    But it is no coincidence that this is not officially accepted as a contradiction to the theory... firstly there are variants on the theory, secondly - as we said - it is always better to take such things in a careful context.

    In the same way, the so-called "Bohemian" has the advantage of being able to adapt to relationships, link:
    http://xxx.lanl.gov/abs/0811.1905

    In short, the "Bohm" theory did not show an advantage in prediction over "Copenhagen", but it cannot be said that it was contradicted either.

    Many of those who prefer it, do so for reasons similar to mine - a more coherent point of view - which of course is already something individual.
    And I think that because the theory is "out of order", there is a tendency to see it as more delusional.

  16. Shmulik,
    I tried twice yesterday to send my answer (fortunately I save in memory before each sending so it won't be deleted...)
    I guess we had a "silence" - courtesy of Abed...
    Trying a name change, maybe it will work...
    First - copy-paste the question to what I wrote:
    ————————————————————————————————————
    Shmulik,
    There are several parts to the paragraph you quoted.

    The first part says that for many scientists the theory is insufficient.
    This is true, and there are several reasons for this, but they are not presented below. At the end of the response I will add what does cause skepticism.

    The second part is from a sentence that, paraphrased, says - if there is no direct indication of the existence of hidden variables - their existence can remain philosophical - but it is possible that a possibility will be found to get an indication of their existence.

    The third part talks about computational practicality of the theory. In general, there is a calculation that requires assumptions such as a finite size for the universe.

    The fourth part talks about meanings that arise for quantum computers. I assume that you use the assumption that the universe is finite, and when you take into account the formulas described in the third part, you get that there is a theoretical limit to the accuracy that can be obtained (note - the third part assumed a finite size in order for the calculations to be finite; this part concludes that if the size of the universe is finite, then a computational solution can accurately solve , and therefore there will be a limit to the theoretical accuracy).

    That is, to your question: the universe sets a limit - but not to the correctness of the theory of hidden variables. Given that the description of hidden variables is correct, the universe sets a limit to the precision that can be calculated with a theoretical quantum computer.
    To illustrate - even classical theoretical computers are limited in all kinds of parameters given a finite size to the universe.

    That is, there is no argument for or against here, there are inferences about the computational limits.
    If someone really wants a computer the size of the universe to be able to do calculations with an accuracy of over 10,000 digits, then they have to hope that there are no hidden variables, but that there really is inherent randomness.

    ———————————————————————————————————————————————–

    The predictions of Copenhagen and "Bohm" intersect in such a strong way that as long as there are no differences, they can be viewed as different descriptions of the same phenomenon.

    However, there is a claim that they managed to find an effect that would give an indication of the existence of the hidden variables (as hinted in the second part that may exist).
    The effect is related to the experiment of photons in two slits, and its results should contradict "Bohm's" prediction. Link:
    http://arxiv.org/abs/quant-ph/0206196

    But it is no coincidence that this is not officially accepted as a contradiction to the theory... firstly there are variants on the theory, secondly - as we said - it is always better to take such things in a careful context.

    In the same way, the so-called "Bohemian" has the advantage of being able to adapt to relationships, link:
    http://xxx.lanl.gov/abs/0811.1905

    In short, the "Bohm" theory did not show an advantage in prediction over "Copenhagen", but it cannot be said that it was contradicted either.

    Many of those who prefer it, do so for reasons similar to mine - a more coherent point of view - which of course is already something individual.
    And I think that because the theory is "out of order", there is a tendency to see it as more delusional.

  17. Shmulik,
    I tried twice yesterday to send my answer (fortunately I save in memory before each sending so it won't be deleted...)
    I guess we had a "silence" - courtesy of Abed...
    First - copy-paste the question to what I wrote:
    ————————————————————————————————————
    Shmulik,
    There are several parts to the paragraph you quoted.

    The first part says that for many scientists the theory is insufficient.
    This is true, and there are several reasons for this, but they are not presented below. At the end of the response I will add what does cause skepticism.

    The second part is from a sentence that, paraphrased, says - if there is no direct indication of the existence of hidden variables - their existence can remain philosophical - but it is possible that a possibility will be found to get an indication of their existence.

    The third part talks about computational practicality of the theory. In general, there is a calculation that requires assumptions such as a finite size for the universe.

    The fourth part talks about meanings that arise for quantum computers. I assume that you use the assumption that the universe is finite, and when you take into account the formulas described in the third part, you get that there is a theoretical limit to the accuracy that can be obtained (note - the third part assumed a finite size in order for the calculations to be finite; this part concludes that if the size of the universe is finite, then a computational solution can accurately solve , and therefore there will be a limit to the theoretical accuracy).

    That is, to your question: the universe sets a limit - but not to the correctness of the theory of hidden variables. Given that the description of hidden variables is correct, the universe sets a limit to the precision that can be calculated with a theoretical quantum computer.
    To illustrate - even classical theoretical computers are limited in all kinds of parameters given a finite size to the universe.

    That is, there is no argument for or against here, there are inferences about the computational limits.
    If someone really wants a computer the size of the universe to be able to do calculations with an accuracy of over 10,000 digits, then they have to hope that there are no hidden variables, but that there really is inherent randomness.

    ———————————————————————————————————————————————–

    The predictions of Copenhagen and "Bohm" intersect in such a strong way that as long as there are no differences, they can be viewed as different descriptions of the same phenomenon.

    However, there is a claim that they managed to find an effect that would give an indication of the existence of the hidden variables (as hinted in the second part that may exist).
    The effect is related to the experiment of photons in two slits, and its results should contradict "Bohm's" prediction. Link:
    http://arxiv.org/abs/quant-ph/0206196

    But it is no coincidence that this is not officially accepted as a contradiction to the theory... firstly there are variants on the theory, secondly - as we said - it is always better to take such things in a careful context.

    In the same way, the so-called "Bohemian" has the advantage of being able to adapt to relationships, link:
    http://xxx.lanl.gov/abs/0811.1905

    In short, the "Bohm" theory did not show an advantage in prediction over "Copenhagen", but it cannot be said that it was contradicted either.

    Many of those who prefer it, do so for reasons similar to mine - a more coherent point of view - which of course is already something individual.
    And I think that because the theory is "out of order", there is a tendency to see it as more delusional.

  18. oak,
    The question of intelligence is actually the question of free will, which is a question that may be impossible to answer, certainly now with the combination of information processing. Regarding free will, you can get an impression of the problematic according to the following link:
    http://en.wikipedia.org/wiki/Free_will

    The only reason we can ask this question is because there is no Aristotelian explanation for quantum mechanics and the 2-slit experiment (as you mentioned). Can we ask if the stone has intelligence because we cannot predict exactly where the grief will fall if we throw it from the building? Can we claim that the planets have intelligence because we cannot solve the three-body problem with infinite precision?

    Intelligence should therefore not be attributed to an electron, certainly not according to the usual definition of this concept (which is certainly ambiguous, since when does a pile become a pile) and this question is one of those questions to which an answer such as, the electron has intelligence, is an answer that is God of the Gaps

    Regarding hidden variables, I would appreciate it if you would refer to what I asked in my previous post and I will save the search and simply paste my question here:

    I read again about the nonlocal hidden variables theory and found an interesting point that may indicate the weakness of this theory
    From the Wikipedia entry: http://en.wikipedia.org/wiki/Uncertainty_principle

    While it is possible to assume that quantum mechanical predictions are due to nonlocal hidden variables, and in fact David Bohm invented such a formulation, this resolution is not satisfactory to the vast majority of physicists. The question of whether a random outcome is predetermined by a nonlocal theory can be philosophical, and it can be potentially intractable. If the hidden variables are not constrained, they could just be a list of random digits that are used to produce the measurement outcomes. To make it sensible, the assumption of nonlocal hidden variables is sometimes augmented by a second assumption — that the size of the observable universe puts a limit on the computations that these variables can do. A nonlocal theory of this sort predicts that a quantum computer encounters fundamental obstacles when it tries to factor numbers of approximately 10,000 digits or more; an achievable task in quantum mechanics

    From what I understand (and indeed not too much), there is a physical limitation that the visible universe places on the hidden variable theory from here, which is a weak point for this theory. is it so

  19. A believing Jew
    The question of whether intelligence can be attributed to a particle is a very interesting question and very related to how you define things.

    I will bother a bit in the hope that it will be of interest. (The second section directly answers the intention of the original question...)

    1) What do you mean when you say that a particle has intelligence?
    I guess you would agree that humans are intelligent, but is intelligence a yes or no question?
    It is quite clear to us that there are levels to intelligence. For example differences between humans, but it is clear to us that different animals exhibit different levels of intelligence.
    So it is said that a mouse is less intelligent than a monkey. And what about a lesser bird. And a fish? Cockroach?
    Can cockroaches be attributed to intelligence? There are actions he takes that keep him alive. But where do you draw the line between automation and intelligence?
    There is a point of view that says that when you go down with the complexity, the intelligence goes down - but it exists.
    They continue the argument towards single cells, and from there to viruses.
    A virus is a huge molecule that actually has a legality that allows it to preserve itself (and even replicate....)
    From here one continues to molecules that manage to remain stable,
    And hence for the particles whose properties allow them to exist (many particles cannot exist, and they collapse into other particles or "cancel" into energy with reversed particles).
    So for our eyes - I have already come across points of view that say that intelligence can also be attributed to particles - even though there are trivial things - the legality that allows a particle to exist...

    2) Does the particle "know" that it is being watched?
    As you can understand from my previous comments - I do not believe that measurement has meaning, and I tried to show how it is possible to interpret cases in which "Copenhagen" gives meaning to the intention of measurement, as effects that arise in a way that is independent of the existence of an observer in the "Bohm" interpretation.
    If I had to explain it from the Copenhagen point of view, I would say that it is not that the particle knows. I would say that it has a form of existence that is different from our particle intuition. Regarding most experiments I would say that when there is an interaction - it forces a collapse into a particle reality.
    There are the more extreme parts, which attribute meaning to the measurement intention of the observer. This is where I'm having trouble - I can't reconcile this point of view coherently, so I can't try to explain...

  20. Here is proof that the Jews tried to deceive the world in all areas for years.

    Your Einstein was a fraud who developed atomic bombs against Islam and attached to him theories that say they are true but they are not and there is proof that they are not so that is why he is not such an Einstein.

  21. As in Disk on Key, which consists of atoms arranged in such a way that their physical state "remembers" the information you copied into it, but I'm not really an expert in the field either, and I asked the opinion of those who do

    Regarding the tunnel, it is of course subatomic particles and not a ball such as ping pong

  22. Shmulik, I don't understand how the universe places a limit on the theory of variables.
    What about the universe and the theory of variables?
    Regarding my question for an experiment with the 2 slits and the interference of the particle wave depending on its measurement/viewing from the side
    It is still not clear how the particle behaves as an intelligent entity that "knows" that it is being watched, after all it is an inanimate particle.
    The responses here further sharpen the magnitude of astonishment and curiosity.
    I read about another interesting field called: "quantum tunneling"
    For example, if we throw a ball at a wall, it has a chance to go through the wall and appear on the other side, it is not clear how, but it is still interesting.

  23. oak,
    Thank you for your explanation.
    I read again about the nonlocal hidden variables theory and found an interesting point that may indicate the weakness of this theory
    From the Wikipedia entry: http://en.wikipedia.org/wiki/Uncertainty_principle

    While it is possible to assume that quantum mechanical predictions are due to nonlocal hidden variables, and in fact David Bohm invented such a formulation, this resolution is not satisfactory to the vast majority of physicists. The question of whether a random outcome is predetermined by a nonlocal theory can be philosophical, and it can be potentially intractable. If the hidden variables are not constrained, they could just be a list of random digits that are used to produce the measurement outcomes. To make it sensible, the assumption of nonlocal hidden variables is sometimes augmented by a second assumption — that the size of the observable universe puts a limit on the computations that these variables can do. A nonlocal theory of this sort predicts that a quantum computer encounters fundamental obstacles when it tries to factor numbers of approximately 10,000 digits or more; an achievable task in quantum mechanics

    From what I understand (and indeed not too much), there is a physical limitation that the visible universe places on the hidden variable theory from here, which is a weak point for this theory. is it so

  24. Israel,
    Ada brought up an interesting point that seems important to me. EPR talks about electron and spin, but the Aspect experiment and similar ones were conducted on photon and polarization. Do you know of an experiment that was actually conducted on an electron and spin? Not that it should dispel the mystery, but from such an experiment we might be able to learn new things about the nature of the electron.

  25. Zvi, thank you.
    It is clear that the analogy I brought is not successful. I have many more of these up my sleeve (which Israel calls alternative physics), but it's a shame to bother. All the abstract models and thought experiments that do not allow refutation tests only raise more questions, more than the questions they claim to solve.
    I will join Ada's greetings: Happy New Year

  26. Hello Ada, your words are beautiful.
    Despite everything that has been said, and surely more will be said, as far as the intellectual discussion concerns physics we feel in complete darkness. Every now and then the light of some Maxwell or Einstein shines just to direct us to another dark path and to a new mystery, greater than all its predecessors. We believe in math and logic, and we have rules of thumb that work well on paper in everyday life but not in physics. We learned from Einstein that two clocks lag behind each other in relation to each other and from Israel Shapira (who quotes without bringing salvation to the world) that the past can be rewritten not only in history books but also in physics laboratories.
    In my intuition, a picture emerges of some elaborate medium (not Maxwell's simple ether) that conducts everything (including light, matter, and possibly also information), and it is not impossible that this medium itself "modulates" an earlier medium, but other than saying with a wave of hands that I am convinced of its existence , all I can contribute to the discussion is only strange analogies and mainly new questions.
    Therefore, I was surprised to hear that someone here understood my words. It is a refreshing innovation.
    Good sleep and a guarantee to you too 🙂 Thank you ♥

  27. Hello Israel and everyone,
    Thanks for your 2 answers. And thanks for all the referrals.
    The first chapter of the lecture at Stanford did not answer the problem that was bothering me. But I will look at it more than half-heartedly in the future.
    I think I visited most of the links mentioned in the answers (at least those that were accessible), I expanded my horizons a little, and below are my conclusions and my doubts as well. Correct me if I'm wrong.

    1) Intertwined particles is a fancy phrase to describe the creation of only one superpositional wave, by scientists. He has 2 heads, (like Dr. Doolittle's Dehikslik). Since it is only one superposition, any measurement will cause the collapse and the end of the experiment. Each additional measurement essentially starts a new experiment.
    2) Not only humans create entangled particles for their own purposes. The universe also has intertwined particles. We don't know how they were created. Probably not easy to discover them either. It is possible that the discovery will lead to the collapse of their wave function and the destruction of such a superposition.
    3) According to section 1) we have a single hybrid product - one and only superposition. Therefore there is nothing to ask about simultaneity. Simultaneity only applies when there is more than one. As soon as there is a superposition wave - the particles run out.

    4) Your second answer: According to one of Schrödinger's scholars, the collapse of the wave function means the end of the experiment (this is not taken from what I read today). That is, today's measurement, after a year, is a new experiment. If you understand it as a continuation of an experiment from a year ago for health. I understand it as a new experiment, and it is clear that it will determine or describe initial conditions related to it. This does not mean that these were the conditions a year ago. So we didn't measure. If we had measured then, the experiment would then have ended with the collapse of the wave function).
    5) A partial description is the one that creates problems. It is similar to the macro world. And it seems that this is why there are conflicts with Einstein and his teachings.
    6) The universe is one perfection (holistic). All of it - including the parts of the universe that we will never be able to observe. Intertwined particles created in the universe, on the other hand, can, in my opinion, extend, as a superposition whose head is in areas outside the realm of the visible universe, and whose tail may reach the earth. Is it possible to have some practical application for this one day? I really don't know what the answer is, but the idea is interesting.

    So thank you Israel, and Yuval.
    I think I wised up a bit, and the clarification was certainly interesting.
    And here it is definitely the place to point out that the article that led to this correspondence was particularly fascinating. Thank you very much for it.

    Happy New Year from Toronto.

  28. Here is Ada.

    You write "I also have no problem with the memory you mentioned of the electron regarding his choice made a year before (it's just puzzling)."

    The point is not the memory of the electron. The point is that his measurement in the present determines the choice he made in the past (did he go through only one crack or through both together? Is he a wave or a particle?).

    And so it turns out that it is apparently possible to influence the past from the future. According to Wheeler's delayed choice experiment.

  29. Here is Ada

    The proof you are asking for can be provided in several levels of detail and difficulty. The proof is actually mathematical - here Bell's inequality theorem is used - but there are also "light" proofs.

    In my opinion, the most relevant link that includes all the mathematical formalism can be found at

    http://www.youtube.com/watch?v=0Eeuqh9QfNI

    Yuval loved the lectures there, and so did I. Let me know if that's enough, or maybe you prefer something simpler.

    Voice of Israel.

  30. Hello Israel and everyone,
    Still no one answered me about simultaneity in entangled particles. (Yuval offered an analogy).

    You wrote: "In every simple, lovable, everyday electron whose wave function collapses immediately upon measurement, the bitter news of the collapse is transmitted to the entire universe immediately."
    Please prove that really, the news (about the collapse - which is the news about the end of the experiment) converted goes to the whole universe. (Which? visible? observed? parallel universes?). And please do not specify which mathematical formula. I would love to hear the proof explanation.

    When I studied the requirements of the EPR experiment it was indeed about electrons (to the best of my memory). On the other hand, all the proofs of the mistakes or "mistakes" of Einstein Podolsky and Rosen were made with photons. For me, it is unacceptable that electrons are exactly the same as photons. Namely: a photo-electron (photon) is just one type of electron, which has its own separate properties.
    Glad you are also talking about electrons.

    I have no problems with the different interpretations of galaxies. I also have no problem with the memory you mentioned of the electron regarding his choice made a year before (it's just puzzling). We still don't know everything about the waves + particles and the universe in general (select the desired universe type).

    And as promised, I follow for refresher purposes at least, the links accessible in the article and in the answers.

    All the best from Toronto.

  31. It seems to me that we are drifting a bit into alternative physics and it might be better if we switch to free responses.

    However, the issue of instantaneous transfer of information is not only characteristic of entangled particles but exists in every quantum object. In every simple, lovable, everyday electron whose wave function collapses immediately upon measurement, the converted news of the collapse is instantly transmitted to the entire universe.
    In the famous two-slit experiment, if we place a detector in front of the background that will tell us which slit the electron passed through, we will lose the interference image even if the detector is placed a light year away from the slits, and at least a year has passed since the electron had to choose whether it is a wave or a particle. On the other hand, if the detector is turned off, the interference image will remain. So then, how does the electron know how it chose at the time whether it is a wave (interference) or a particle (no interference)? Doesn't this have an effect on the past from the future?

    Therefore, the matter of the hard middle does not seem related to me (Yuval). This has nothing to do with polarities. But if you want, you can switch to freedom.

    Israel, West Tel Aviv.

  32. jubilee,
    From Maxwell's equations (which predicted the existence of magnetic AM waves and thus gave an explanation for light) it is clear that there can be no longitudinal electromagnetic waves, since the direction of the electric field E and the direction of the magnetic field B are always perpendicular to the direction of the wave's progress.

  33. oak
    Can "intelligence" be attributed to a particle that behaves differently when it is observed or measured?
    It seems as if the particle "knows" that it is being watched and changes its behavior

  34. Hail Israel, Hail Jubilee and Hail to all,
    I promise to come back and refresh myself on all the links in the article and in the comments. If I have something to say later I will write separately.
    In the meantime, it seems to me that the concept of intertwined particles also hides the idea that the particles are in one frame of reference (and therefore the simultaneity that is avoided - guaranteed in advance). However, there can always be additional reference systems, and according to Einstein there is no longer such a phenomenon called simultaneity. It doesn't matter what they hide behind in order not to use it.
    If things happen at the same time - no matter what the reason - this is simultaneity.
    According to Einstein, simultaneity no longer exists, because there can always be additional frames of reference.

    And actually - beyond the fact that nowadays it is accepted that in entangled particles the final phenomenon happens simultaneously in 2 places, without passing through space, and without passing through time, and without passing through information - how many real attempts have been made to make sure that this happens?
    And how do you prove simultaneity if there is no such animal? (There is such a word but there is no such phenomenon).

    Yuval - The question is interesting, but my knowledge of waves does not allow an answer of any kind. Regarding earthquakes, different waves will travel on different "tracks", probably due to their nature. And there really is a difference between the arrival times of the shear waves compared to the initial waves whose names I have forgotten. But in any case, I'm not sure that the reality I know about earthquakes, coincides with your examples of longitudinal and latitudinal waves.

    Happy New Year to all of us.

  35. Israel d'California (there are more Israelis in Los Angeles than in Tel Aviv. Right?),
    As I mentioned in the evening and the evening, we still do not know how the middleman works (and if there is one at all, but we will now assume that there is). Let's compare it to a rigid tube with a liquid flowing inside it: the liquid moves inside the tube at a known speed c; But the tube can be rotated, and since it is rigid, it rotates all at once. And the analogy: the polarization of the photon is not a "hidden variable" but depends on the polarization path that is directed to it. The rotation of the polarizer on one side of the experimental site affects all the polarization paths in the environment, and the information about the change travels at an infinite speed because the polarization paths are rigid.
    However, as we remember, we do not know how the medium works. I suggest addressing this issue before we make far-reaching assumptions.

  36. Canada, Scotland Yard,

    There is no question of faster or less. The quantum link is instantaneous, in zero time or infinite speed. If it is not clear why this is required, even after looking at the link to Nick Herbert's proof or Susskind's lectures, point it out.

    This is Israel.

  37. Canada and Israel and all those interested,
    Please give your opinion on the following analogy:

    We know, for example from earthquakes, the phenomenon of longitudinal and latitudinal waves that are created simultaneously. Longitudinal waves are faster than latitudinal waves even though they move in the same medium.

    We know that electromagnetic waves are transverse waves.
    We did not find electromagnetic longitudinal waves, but if there were such, then from this analogy we would expect them to progress [much] faster than the waves we know.

    there is what to talk about?

  38. Israel Shapira and others who are interested. Hello.
    I have a problem with the determination regarding the feature that passes immediately. (It doesn't matter to me the distances Jerusalem-Haifa or Tel Aviv-Yirah for example).
    I rely on Einstein's assertion that there is no simultaneity. That is: immediate = simultaneous. And there is no such thing (special relativity - the possibility of different reference systems).
    Let's assume for a moment that the feature is passed one by one. First Jerusalem then Haifa, (Jerusalem one spin. Haifa: its reverse) and only our presuppositions say it was simultaneous.
    Which features can pass one after the other and which can't?
    And if certain features pass one after the other - maybe this tells us something different than we thought before.
    What do you think? what do you think?

  39. Yes and no.

    To understand the subject of interweaving, think about a coin being thrown. If you check, you will see that it only falls on a tree or on a field. But what happens when it is still in the air before the test? It is found in the "superposition" of a Vefeli tree. The same with a spinning top or a cube, which before the test are in a superposition of all states.

    In interlacing, if two coins are interlaced, then if one falls on a tree, the other always also falls on a tree. The same with entangled photons, which always measure the same polarization, or electrons, which always have the opposite spin.

    How does this happen? Einstein said: hidden variables. They were designed that way even before they broke up. However, according to quantum mechanics there is no such thing as hidden variables, and only the very measurement determines which state will be chosen. Like with a coin or a spinning top. Therefore, as soon as you have measured one electron/coin/spindle in Jerusalem and found its condition, the attribute is immediately transferred to its brother in Haifa.

    So who is right?

    For a relatively simple proof of Bell's theorem showing that quantum mechanics is correct and non-locality does exist, see:

    http://quantumtantra.com/bell2.html

  40. Yes and no,
    Although I didn't go into detail, there are a number of experiments that make people accept the Copenhagen interpretation as reasonable.
    I will try to present things as best I can, and show that there is another interpretation - and why it is my favorite.

    Reasons that tend to accept the Copenhagen interpretation:

    1) Bell's inequality - a concrete example: there are two photons conjugated in polarization. Each of the photons is sent to a system that emits one of three polarizers - one at 0, the second at 120 and the third at 240 degrees.
    Both systems trigger independently, and save for each measurement whether the photon passed or not.
    When you see that both systems have the same polarization - throw away the measurement (then there will surely be a 100% correlation...).
    It turns out mathematically that a match is expected (as to whether the photon passed or not) in 1/3 of the cases. (or according to the inequality at least 1/3).
    In practice you get an adjustment of 1/4. And this is also what the quantum mechanics predicts. And this supposedly contradicts the idea of ​​hidden variables. And in this case it means that the polarization could not be predetermined, since it should be limited to the described inequality. From this it follows that the very experiment - forced the photon to choose a polarization.

    2) The famous two slits experiment - with a slight change: on one of the slits - you don't put a normal gauge, these are "indirect gauges" - I don't know the details, but if you put the gauge on a single slit - the particle passes, and you can still conclude from the gauge that it is Passed (I think it's a measurement of an electric field that an electron creates).
    But when you put the gauge on one of the two slots in the famous experiment - there is no struggle. That is, as it were, your very ability to measure whether the electron passed through one slot or the other - prevents you from getting the expected interference pattern.
    This is one of the most convincing things: it turns out (so to speak) that when an observer looks - the particle behaves differently.
    And as scientists - if this is the case - they accept the theory. Even if it turns out that there is an almost magical effect to observation. Metaphysically, this is a very non-trivial assumption...

    3) Tradition. I know it may sound strange, but when the majority of the community already accepts a compromise, they tend not to go against it. After all, mathematically it works. And those who were brought up according to the Copenhagen point of view, tend to see physics as a tool designed to describe what the observer sees, and since the results are correct, there is no need to change the point of view.

    ——————————————————————————————————

    After all this, it is said that there is a "De Broglie - Boehm" interpretation.

    Broadly speaking, "Copenhagen" looks at the Schrödinger equation as a description of the chance of finding the particle in a certain place, and as long as there is no measurement, it can be said that the particle behaves like a wave - according to the Schrödinger equation.
    "Bohm" says that the Schrödinger equation describes a real wave, which carries the particle. This is called wave-pilot. That is, the particle rides on this wave.
    Just what - the configuration of the wave is determined from the whole space - at once. That is, it is not local.
    (We tend to present this as a mental compromise - but remember that "Copenhagen" is not local either).

    Now what's the matter? The very existence of the particle as a physical body that rides this wave is a hidden variable.
    So does Bell's inequality contradict it? No!
    First, it is quite clear that it is not, because his math is the same in terms of measurements as "Copenhagen", that is, here too the Schrödinger equation is used for the solution, and you get the 1/4 adjustment to the experiment I described at the beginning.
    But did we say that inequality does not contradict the idea of ​​hidden variables? So the point is that because the wave on which the particle rides is determined from all space in a non-local way - this is not contradictory. The inequality contradicts only hidden variables in a local description.

    And what about the experiment with the indirect measure?
    Here too, they showed that the very invention of the indirect gauge in the second slot - changes the shape of the wave in such a way that the wave behaves as if the slot is blocked. A bit like electricity - if you have a closed circuit and you put a resistor at the same time (even a very weak one) the electricity will only pass through the wire without the resistor. This is only an analogy - for those who were bothered by it, please ignore it.

    In the end, this is a theory that requires - at least from me - less "mental compromise". It has a description of reality independent of the observer, it has hidden variables that make it possible to say that at the deep level there is really no lottery, you don't look at a particle sometimes as a particle, and sometimes as a wave, and most importantly - there is no almost mystical meaning to the act of observation; That is, from a metaphysical point of view, it allows to see the existence of an objective reality!

    And the way I see it, the main reason that it is not the theory accepted by the majority, is mainly things related to "academic tradition".

  41. Alon, thank you very much for your response, great summary and closure of corners
    From what I understand in the end, there is no physical proof of the measurement effect, but only a philosophical hypothesis.
    A hypothesis that is indeed based on a lot of quantum laws and quirks, but still a hypothesis
    If the electron's oscillations are random in one way or another, there is no reason why the same randomness would not be present without measurement.
    The Copenhagen interpretation seems to me to be a "surrender" to our hidden and wonderful reality, what? If you don't know how a wave is created, they say there is a god named Poseidon? Or continue to investigate until you find out that these are the fluctuations of the moon and the earth?
    To say that the Torah is a complete Torah by clinging to philosophy seems to me a bit... not a good smell.

  42. Alon,
    You can provide a personal email address that you can contact for questions and inquiries
    Thanks

  43. The border between physics and philosophy (after hundreds of years of separation) - simple (and) fascinating, thanks Alon.
    And thanks also to Idan for the fascinating article in itself.

  44. Yes and no,
    Your question touches on a very sensitive point. This is the border where physics meets philosophy (after hundreds of years of separation...).

    I will try to refine the points of view on the subject according to my understanding.

    1) Why should reality be read?
    Is there even such a thing as objective reality, where every point of view "samples" it?
    If it exists, is its true description in the objects we can describe?
    Maybe reality is only what you see - right now?

    2) What is the purpose of physics?
    Does it describe the same objective reality?
    Then maybe its purpose is to give an explanation _why_ things happen the way they do?
    Or maybe it should describe the experience of your point of view only?
    And then its purpose is to _describe_ how things happen?

    As I understand it, the relativistic approach tends to look at reality as something that exists objectively, and that does not depend on a "sample" or an "observer".

    The quantum approach (Copenhagen) tends to look at reality as something that has meaning given an "observer". Which means that her description focuses on how the "observer" sees things. From this point of view there is no "meaning" to try to describe objective reality. Or another way of saying it - for them, physics deals with describing the point of view of the "observer" only.
    Let's take for example the experiment with entangled particles.
    There is "observer0" who made sure to pair the spins, and sends them to two different observers. He knows they are conjugated, but does not know which one is up and which is down.
    "observer1" measured one of the particles. He has given knowledge (received/coordinated with observer0) that there is another conjugated particle, which was sent to "observer2", and therefore he can conclude (only together with observer0) that if he himself measured up the other will measure down and vice versa.
    "observer2" is of course symmetrical to "observer1".
    The problems start when you try to look at the Copenhagen interpretation from an objective point of view.
    According to our description, it is clear why "observer1" knows instantly what the measurement that "observer2" will measure will be.
    But does because he knows, then does that mean that there is an objective point of view, which also says the same thing?
    According to the Copenhagen interpretation - there is no reference to an objective point of view. She doesn't know how to respond to this question, or if you want - it's a question that simply has no meaning for her. It is not the right tool - it is like trying to measure a temperature with a ruler.

    And by the way, as I understand it, the way to "reconcile" with relativity is related to the speed at which "observer1" and "observer2" can transfer information between them.

    By the way, this makes it possible to look again at Schrödinger's cat experiment:
    If it is said that Schrödinger's cat is "observer1" and the man doing the experiment is "observer2", then of course the cat knows whether it is alive or dead. But since there is no communication, the experimenter looks at reality only according to his knowledge with the quantum machine, and when he calculates the expected results, he is blocked from a statistical solution. (The same superposition of half alive, half dead).

    When he checks and sees that the cat is alive, does it change anything? Not for the cat, but yes for him.

    But what if you can't always check if the cat is alive?
    For that matter, let's think of one particle as a system that has two boxes, in each a cat and a peephole.
    The problem is that the boxes hide each other, and you can only look properly through one window in each experiment.
    Now, although you have the option to "call" one of the cats and see if it's alive or dead,
    You are forced to calculate statistics only for the other cat.
    This is roughly the principle behind the uncertainty that exists regarding the position-momentum of a particle.

    The only problem is that I am still describing things from an objective point of view. I said there are cats, and from my description it is clear that there is an objective reality in which it is possible to say whether each of them is alive or dead. And Einstein tended to see the world that way. That is to say that there are hidden variables, and that it is meaningful to describe an objective reality that tells whether everyone is alive or dead.
    I didn't go into the depth of Bell's inequality, but the claim is that he was able to show that statistics contradicts the hidden variables approach. That is, it cannot be said that there is an objective point of view that will see the two cats.

    And the balconies regarding the question of the moon: according to Copenhagen, it can be said that as long as you do not see the moon, "there is no meaning" to the question of whether it is there. (There is only meaning in making a measurement, and saying yes or no. The rest is not related to physics - as Copenhagen sees physics).

  45. Hi friends, I enjoyed the article and the comments even though I didn't understand anything...
    Well maybe a little, vaguely...

    And yet, how is it possible to claim that measurement determines reality? And not just exposing her?
    If I had not measured - the particular case would not have occurred? Not exactly the same way?
    Albert was right when he asked if the moon exists even with closed eyes..
    Let's claim that reality is random and not deterministic, but where is the connection to measurement?

    Can someone explain the basis for the claim that the measurement determines/creates?
    again…
    Claiming that reality is random/deterministic/local/non-local/good/bad/divine/coincidental/hoga boga is all well and good, but where is the basis for the claim that the measurement affected reality? based on what?

  46. sympathetic,
    I don't remember if you are for Tevech or against him. If yes, does the information that passes through the experiment (and it is not light) require a different medium than the one that transmits the light? If against, then forget it

  47. Israel

    I'll try one last time because I'm getting tired. For Einstein there was no difference between the transmission of information
    For transmission correlations the speed of light was the upper limit therefore anything that exceeds the speed of light
    For him, it is a contradiction to a private relationship. The cleverness that only information is not allowed to pass the speed of light is late and is intended to settle the paradox. The theory of relativity as a principle does not talk about information

    jubilee
    An element with atomic number 79 is a heavy element

  48. Again, I just want to point out that many of you responded to Shmloik as Shmolik but Shmolik is not Shmolik he is me and by me I mean the one who responded to the example here:
    https://www.hayadan.org.il/satellite-measurements-and-global-climate-models-closer-090512/comment-page-1/#comment-359353

    Now he has corrected himself and calls himself Shmulik, but again, not me.

    In any case, the article is great and I would be happy to receive a link to the amazing article by Roderich Tomolka from 2006.
    Is it: http://www.maphy.uni-tuebingen.de/members/rotu/papers/grw2.pdf
    או
    http://www.maphy.uni-tuebingen.de/members/rotu/papers/ssr.pdf

    of
    http://www.math.rutgers.edu/~tumulka/papers/

    Thanks

  49. Thanks to the explainers, except for Alon who is uplifted from the people, and who is uplifted.

    So far no intuitive explanation has been given, in particular that the speed of the electron around the atom is unknown.
    On the contrary - the explanation is that it does not exist for a long time but is constantly ionized and regenerated in different places around the atomic nucleus with a high probability of finding it in an orbital that corresponds to its energy level.

  50. sympathetic

    I am aware that Einstein did not know about Bell's theorem. I wrote it in my first comment.

    My point: even without Bell's theorem, there is no contradiction between non-locality in quantum entanglement and relativity.

    Non-locality, wonderfully strange and fantastic as it is, does not contradict relativity even though there is no doubt that the spin information passes instantaneously from electron to electron. If it turned out to happen at the speed of light alone, it would still be surprising but not paradoxical. The apparent paradox is in the passage of information faster than light.

    However, as you said, "he did not make the distinction between information and correlations". Otherwise he would have come to the conclusion that there is no paradox here, even just with the knowledge he had in 1935.

    Because this is the important point: the fact that information passes, still does not require that information can be sent at a speed higher than light, and only this is forbidden by relativity.

    It is not possible to send information through interleaving. No matter how fast. With or without Bell's trial.

    So why is it called a paradox?

    Just a lack of attention to detail. That's my point.

    Unless, as I wrote: "If Einstein was alive when Bell published his inequality theorem, the old fox would have found some creative way out of the tangle."

  51. Israel

    Your question is not logical but a question about Einstein and it has already been answered. Einstein did not know
    the Bell inequalities and therefore the thought experiment came to show that Bohr's argument that quantum theory
    is a complete and final theory and it is not a partial description that will be replaced by a more basic theory is wrong (the argument
    of Bohr).

    According to Einstein for his time, the EPR experiment could be given two interpretations, one has a basic theory
    Moreover, quantum theory is not a complete description of nature, the second possibility has a more basic description
    The so-called hidden variables are the ones responsible for the coordination between the particles. For Einstein, nothing
    cannot travel faster than the speed of light he did not make the distinction between information and correlations and therefore
    He came to the conclusion that quantum theory cannot be the complete buffalo of nature.

    jubilee
    Fiscal theories in general are not limited to length or time scales. The theory of relativity is not a theory of the macro. When calculating the electronic levels of the gold atom, relativistic corrections must be introduced to get a good comparison with the experimental ones, the reason is that gold is a heavy element and therefore the outer electrons move at relative speeds

  52. Shmulik, Alon and Shagyi

    The intuitive explanation given by Alon regarding the stability of a hydrogen atom is good but not enough.

    In the simplest description, the electron is a point particle that "surrounds" a nucleus with an opposite charge when the force is greater than the radial acceleration (or in other words, balanced by the centrifugal force) - just like a satellite around the earth.
    But this description is incomplete (even for Levin).
    The electron is a charged particle and therefore being accelerated means that it radiates and a naive calculation shows that it had to radiate the full energy stored in its orbit over a period of time of about 9-^10 seconds (a billionth of a second). In fact, this argument is also true in the case of the satellite around the Earth, but here it is gravitational radiation and it turns out that the satellite will not change its orbit significantly throughout the life of the universe.

    The intuitive explanation is therefore not enough.
    In this case, one should resort to the error method and really solve the Schrödinger equation for the Coulomb potential.
    If you insist on intuition, you should say that the electron is forced to stay in certain orbits and therefore "cannot" radiate its energy.

  53. There is no choice but to enter the thick of the beam. We still do not understand the structure and mode of operation of the medium that carries all the phenomena of physics (in fact, it is still not clear if such a medium exists at all). Until then, we can note the following feature: quantum mechanics deals with the micro, while relativity deals with the macro. Not all sentences in statistics that are relevant to one field are relevant to the other field, and vice versa.

  54. Shmulik,
    I tried to give an intuitive feeling to this "power" you are looking for,
    But if you want to understand at the quantum level, we'll settle for Sagi's answer.

  55. philoshit,
    1) Don't know David Albert.
    2) I expected this from my familiarity in the field. The vast majority are adherents of the Copenhagen interpretation.
    3) I tried to flatter, and I came out condescending. It must have bothered you. Ok.

  56. oak:

    You wrote "Kudos to the author of the article" and that you expected to find inaccuracies, etc.
    Why did you expect such a thing? David Albert is a well-known quantum physicist.
    I would say your response is a bit arrogant...

  57. Asaf

    In my understanding, the spin or polarization can be measured without waiting for information from the other side. The state of the polarizer is predetermined and there is no need to move it every time.

  58. sympathetic

    I agree with you that even with a single particle it can be argued that information is instantly transmitted to the entire universe at the moment of collapse, and there is nothing special about interweaving. However, the subject of our article - and also the EPR article - is interweaving. It is also more convenient to deal with it mentally, because we can think of 2 intertwined particles that are held in different places (an exotic example: photons in a condensed state - Einstein that are in different containers, one on Earth the other on Mars) and a collapse in one causes a collapse in the other. In such a case there is no doubt that the two containers are in different and specific places. If one of them is connected to a bomb that is activated by the up mode, the other party will be able to know immediately if the bomb has exploded or not by measuring the spin, long before the transmission from Curiosity arrives. (Assuming, of course, that both of them had previously spoken to check the spin at about the same moment).

    About EPR in Wikipedia it is written Since this implies that one particle is communicating with the other instantaneously across space, ie faster than light, this is the "paradox".

    And that instant communication is the essence of the paradox. If there is no paradox, but only "a simple confusion of concepts" why does Einstein claim that there is a paradox?

    Note that even if your deduction is correct and indeed all we have is "simply confusing concepts" this still confirms what I said: "The question is not physical but logical: how did Einstein not see what we see?".

  59. Good morning everyone.
    For a believing Jew -
    A hydrogen atom has one electron so there is nothing to keep it away from the nucleus of the atom.

    Alon and Sagi-
    I tried to understand your explanation, the fallout for satellites is less suitable since the satellite has an equal radial acceleration but offset by gravity, and the electron does not have a defined movement at all (not even a defined position - but a probability cloud), and an energy level is a description of a situation, not a cause or a force capable of causing something .
    Are there additional repulsive forces between the electron and the proton?

  60. The particle has no speed, the effect is immediate, this means that the two particles that are at the two ends of the universe are actually the same particle, one particle, not two.
    It follows that one particle is in two different places where it is zamit.

  61. Where exactly is the threat? I eagerly read hoping to see some change, some progress, but in vain.
    Show us that information can be transmitted faster than the speed of light.
    Until then, Einstein rules.

  62. To Israel,
    From what I understand (and I'm not sure I understand correctly) you can't measure the spin if you don't know what the state of the other particle is. It is roughly like measuring the angle of a particle and asking if it is 180 degrees from the other particle without knowing the angle of the other.
    Therefore you first measure the spin and then wait for the information of the first particle to be transmitted to you so that you can decipher the results.

  63. Israel

    Before a measurement was made there are no two electrons, there is only an entangled state and no properties of a particle
    isolated and cannot be attributed to particles according to the Copenhagen interpretation. Measurement determines
    The state to which the wave function collapses and is determined instantly in the entire space is not passed
    information.

    I don't understand why you think the problem with the EPR experiment, you could argue about a particle
    Free whose wave function is spread throughout the universe because as soon as it is measured information passes
    For the whole universe, the region where the particle is measured, zero the wave function. According to the Copenhagen interpretation
    Before the measurement, position was not a defined property of the particle, so no information was passed
    There is simply a confusion of concepts here

  64. sympathetic

    According to my understanding, if 2 entangled electrons are a light year away from each other, as long as we have not measured their spin, they are in a state of superposition of "up" and "down" in all 3 axes. As soon as the spin of one of the electrons is measured, it moves to a distinct state of one of the states, and its distant brother immediately moves to the opposite state.

    The information of the spin state (or polarization of the photon) passes from one to the other. Otherwise, how does the remote electron know to choose the opposite direction?

    Einstein's answer: hidden variables. The spins or polarization were already there before and the measurement only revealed what they were. Bell's inequality theorem (true, after Einstein died), proved that this is not the case, and information passes from one electron to another.

    If you can, give an example that will illustrate your point that information does not pass.

  65. I just want to point out that many of you answered Shmloik as Shmolik but Shmolik is not Shmolik he is me and by me I mean the one who responded to the example here:
    https://www.hayadan.org.il/satellite-measurements-and-global-climate-models-closer-090512/comment-page-1/#comment-359353

    In any case, the article is great and I would be happy to receive a link to the amazing article by Roderich Tomolka from 2006.
    Is it: http://www.maphy.uni-tuebingen.de/members/rotu/papers/grw2.pdf
    או
    http://www.maphy.uni-tuebingen.de/members/rotu/papers/ssr.pdf

    of
    http://www.math.rutgers.edu/~tumulka/papers/

    Thanks

  66. Israel

    Einstein was not familiar with Bell's inequalities which were developed in 1964 long after
    EPR

    Regarding the transfer of information, there is a logical contradiction in your words. Information is
    What we know about the world and since the quantum state is not accessible to us
    Before measurement, he cannot transmit information. What passes between
    The particles cannot be information per definition (logical)

  67. It is very interesting to try interweaving over time and in a given place. If given a particle in the past and a particle in the future (that is, two particles that are distant from each other in time), the particle in the past can affect the particle in the future if they are located in the same place.
    And so it is possible to move from the past to the future without being present in the present between the past and the future, without being present in the channel that links the past to the future, like jumping between the past and the future.

  68. Eventually, relativity will fall because of quantum mechanics.
    They will never be able to walk together.

    Einstein's honor rests in his place.

  69. The speed of the carrier wave is also finite. The speed of information transfer in interlacing is infinite.

    And as I mentioned, the question is not physical but logical: how did Einstein not see what we see.

    Looks like we're done. Good day my brother.

  70. Analogy only.
    A carrier wave is "allowed to exceed the speed of light" (the peak of the sine can exceed the speed of light),
    But the speed of the information he modulates - or the "bunch" - is limited to c.
    I think it's very analog. There is a signal that supposedly goes faster than c, but "it's okay" because the information doesn't. But I am me, and you are you.

    And as it seems from the physical aspect, you also gave yourself an answer that satisfies you, so there's really no point in me continuing...

  71. oak

    If you look at the link to Wikipedia, you will see that, according to Popper, Einstein was clearly wrong. This is also the popular opinion.

    If you look at the title of the article, you will see that there are many who hold that non-locality contradicts relativity.

    If you check von Neumann's interpretation you will see that there is no contradiction between non-locality in quantum entanglement and relativity.

    My question: How is it that EPR did not see that there is no contradiction? They wrote that there is.

    The answer I gave: It is possible that they did not get into the thick of the subject.

    but how? After all, Einstein..

    And this has nothing to do with the speed of the group. The question is logical.

  72. Israel Shapira,
    I hope I understand what you are saying, also given the link.

    I will try to briefly describe why I think EPR did not try to resolve the contradiction:

    Einstein claimed that he found a "bug" in the Copenhagen interpretation.
    After they checked and saw that it was really happening, they told him - it's not a "bug" it's a "feature".

    At the time of writing the article, he did not think that the "bug" was something real that he needed to address.
    He thought he had found a fault, which would probably conflict with the theory of relativity - because it is inherently incorrect.

    It's about why it didn't come up in the first place.

    Regarding the solution to the paradox - yes, this is also the solution I know. A signal was sent, but no information.
    If you know, it's very analogous to the speed of the wave, and the speed of the bunch. The carrier wave can exceed the speed of light, but the speed of the group is limited, and "it's okay", because the carrier wave does not transmit information...

    Anyway, in my personal opinion, this interpretation is wrong, and I recommend looking for David Boehm's theory.

  73. lol… 🙂

    I'm really not suitable for formal study. I did graduate with a reasonable grade, but it was despite (and not thanks to) my mathematical skills.

    I'm better at intuition, and it comes for free - but without warranty 😉

  74. oak

    From Wikipedia:

    http://en.wikipedia.org/wiki/EPR_paradox

    Einstein, Podolsky and Rosen asked how can the second particle "know" to have precisely defined momentum but uncertain position? Since this implies that one particle is communicating with the other instantaneously across space, ie faster than light, this is the "paradox".

    So if the maestro claims that the passage of information faster than light is not possible and hence the paradox, how is it that we claim there is no paradox? What do we see that he did not see?

    The answer may be in what I said: the paradox is the result of a sweeping generalization, without going into details. According to relativity, information cannot be sent faster than light. In interlacing, information travels faster than light. Seemingly a paradox. But here are the subtleties: true, information travels faster than light - but that doesn't mean we can send this information.

    A bit presumptuous, so the title and topic of the article are as they are.

    See also:

    https://www.hayadan.org.il/free-speach-20100800/comment-page-36/#comment-356737

  75. oak! Make me a boy!

    Apart from your expectations for inaccuracies - your response is perfect 🙂

    You put a lot of things in order for me - and in general I like your wording...

    Do you take private lessons in physics and mathematics?

  76. First, kudos to the author of the article!
    I expected to find multiple biases and inaccuracies, as can be found even with "quantum mechanics experts".
    Except for a few sentences that I didn't understand, and maybe one that I would have phrased differently - a perfect description - given the scope.

    Israel Shapira,
    As far as I understand, the purpose of the article was to show that the path comes from interpreting quantum mechanics incorrectly, because it requires accepting non-locality.
    That is, they (e+p+r) assumed that it is enough to show that according to the classical interpretation there is non-locality in order to contradict it.
    A "by-product" of this was also a contradiction with relativity, but for them it is clear that there was no need to reconcile this contradiction, because it is clear to them that the classical quantum interpretation is wrong.
    Only in retrospect, when it turns out that the effect does exist, did the need arise to reconcile the contradiction with relativity.

    Shmulik,
    A. I saw a video on YouTube of someone who presented the subject in "google talks" (or whatever they call the documentation of presentations that are given to their employees). The presenter focused on an experiment of polarization of light, and most of the experiments that can be understood on the subject are with polarized light that goes through processes that should eventually decide between the interpretations.
    Although I saw - I was not able to assimilate, and certainly not to explain, but if you look - you will find.

    B. Intertwined particles = particles that have a conjugated property.
    Assume the coupling is to spin. And now if you measured in one up; When you measure the second one you will inevitably find that it is down.
    It doesn't matter when you measure each one - the measurements will always be that one is up, and the other is down.
    The real question (in my opinion) is whether the first was always up and the second down, and "we didn't know it", or whether the act of measurement created a situation that requires the particle to "choose". In terms of the classical interpretation, the particle "has not chosen yet".
    That is, according to the classical interpretation, the two particles "have not chosen yet", but they are conjugated. When one "chooses", the other can no longer choose. In the case of the second particle, the measurement no longer affects, Ella only measures.

    The classical view says that as long as there is no measurement, the particle is like our space, but not exactly. Didn't go into depth, just said that in this state it behaves like a wave, and when there is a measurement, it "collapses" into what we call reality - in which there are particles.

    It is a summary of aspects of the classical interpretation called the Copenhagen interpretation.

    An interpretation that I personally do not like at all, and it looks more like a mathematical tool, thrown without mediation to describe reality.

    David Boehm (mentioned in the article) developed an interpretation that seems to me to be much more physical at its deepest level. Worth being interested and reading.

    third. Sagi described mathematically why, but as is clear from my words, I am more in favor of an intuitive understanding:
    From a simple point of view - why would an electron be attracted to a proton? Because of the electro-magnetic attraction (between plus and minus).
    Why does the electron not fall?
    In the analogy you will provide for now: why don't satellites fall into space? Because they have speed. Similarly, an electron has energy. And when energy is added to it, it moves away to an orbit further away from the nucleus. Add enough energy, and the electron will escape the nucleus completely (leaving an ion).
    just what? Not every energy you add is possible (related to the explanation of an error). Because of constraints, and other mathematics, there are only a few paths that are possible, and only if the energy that is added is suitable for moving to one of these paths - then the electron's jump will stop...

  77. I still don't understand where is the proof of non-locality? Who said that the uncertainty of quantum mechanics is actually our lack of knowledge about all the parameters? And if we know how to measure all the parameters, we will know exactly the result and not have to use statistical tools?

  78. Hitting with a ram, the example of the fist is vulgar, ugly and unnecessary. Much more elegant examples could be thought of.

  79. to Shmulik-
    A. Do an experiment and show that reality behaves as expected from the quantum theory and not as expected according to EPR

    B. In quants, the measurement process itself must be taken into account. Basically, the measurement changes the state of the particle.
    Here too it risks experimenting, when there are two entangled particles, let's say their spin states are entangled, so no matter when and where you measure their spin states, the results will always be matched between the two particles (unlike two non-entangled particles in which there will be a match only in 50% of the cases).

    third. You are invited to solve the Schrödinger equation with a Coulomb potential and see why this happens (hint, an exact solution can only be obtained for a nucleus and a single electron).
    Basically this is a quantum phenomenon, you can think of it this way that for other orbits the electron is supposed to wrangle with itself, destructive wrangling, and in the allowed orbits the wrangling is constructive, so the electron can live there quietly and peacefully. Just remember that this explanation may be beautiful and picturesque but it is just that - beautiful and picturesque, it does not mean that it is really related to reality, the only correct explanation is obtained from the solution of the Schrödinger equation.

  80. Shmulik, I have an answer to section C:

    The greater the distance between the electron and the nucleus, the weaker the attraction, and conversely, the smaller the distance, the stronger the attraction.
    The electron only rotates around the nucleus and does not fall or collapse because there are other forces that attract it - repulsive forces between electron and electron..
    I think there is a Coulomb law that quantitatively shows the forces acting between two charged bodies

    I also didn't quite understand section B. Which cap is it about?

  81. Hello friends
    A- Can someone explain in Hebrew how they managed to contradict the article known as EPR.

    B - If you do parallel tests on the two interwoven particles, at the exact moment, will the result always show that they are indeed interwoven? If so then the great locality will continue to rule the roost.

    b- Why doesn't the electron fall into the proton in the atomic nucleus?

  82. Sorry, friends, but a sentence like "according to non-locality, a fist in Tel Aviv can crush a face in Jerusalem" does not suit you. What's wrong with a different, less violent touch? And even with some kind of positive, human emotion?

  83. In the original EPR article, Einstein and his colleagues claim that quantum mechanics is not a complete theory because quantum entanglement and the non-locality required by the uncertainty principle contradict the theory of relativity, which prohibits the transfer of information at speeds higher than the speed of light.

    According to this argument, there is no doubt that information - the electron spin information or the photon polarization - passes between the entwined particles in zero time, or at infinite speed. Therefore, because of the prohibition on sending information faster than light, we must conclude that those variables were hidden from the beginning in the entangled particles, and the measurement only revealed them.

    And hence the conclusion of the article is that there is no uncertainty principle, and in fact quantum mechanics is incomplete.

    Bell's inequality theorem and Aspect experiments and its successors come and prove that Einstein was wrong: non-locality exists.

    But, reassuring defenders of relativity, this does not contradict the theory. True, they claim, information does pass and passes, but this does not mean that we have the power to send information. If we measure the spin information at our end, we will only find that it is "up" or "down", but we will not be able to tell if we or the other party, the one who sent the entangled particle, caused the collapse of the wave function, therefore there is no possibility of sending information faster than light. This is how relativity took advantage of a large pasture (or maybe not? According to our articles).

    And my question: how is it that Einstein Podolsky and Rosen did not see the exact same thing? After all, in their article they claimed that such a contradiction exists and does exist!

    I don't know about Podolsky and Rosen, but I believe that if Einstein had been alive when Bell published his inequality theorem, the old fox would have found some creative way out of the tangle and continued the eternal debate with Bohr about the validity of quantum mechanics.

Leave a Reply

Email will not be published. Required fields are marked *

This site uses Akismat to prevent spam messages. Click here to learn how your response data is processed.